Help thank you!!!!!!!

Help Thank You!!!!!!!

Answers

Answer 1

[tex] v = \sqrt{4900} + \sqrt{8100} = 70 + 90 = 160[/tex]

Answer: D. 160


Related Questions

CAN ANYONE HELP ME THANKS FOR BRAINLIEST ANSWER PLEASE What one is the standard form of the equation y = – 1/4 x – 2? A. x + 4y = 8 B. x + 4y = – 2 C. x + 4y = – 8 D. –x + 4y = – 8

Answers

Answer:

The standard form of the equation of y = -1/4x - 2 is x + 4y = -8 which is C

Find x and y, please solve with steps and leave answers in fraction form, THANK YOU

Answers

Answer:

Below

Step-by-step explanation:

Using the proprtionality relation:

● 8/10 =5/x

● (4*2)/(5*2) = 5/x

Simplify using 2

● 4/5 = 5/x

Multiply both sides by 5

● (4/5)*5 = (5/x)*5

● 4 = 25/x

Switch x and 4

● x= 25/4

■■■■■■■■■■■■■■■■■■■■■■■■■

Again use the proportionality relation but this time with y.

● 8/10 =7/y

8/10 = 4/5

● 4/5 = 7/y

Multiply both sides by 5

● (4/5)*5 =(7/y)*5

● 4 = 35/y

Switch 4 and y

● y = 35/4

Aiden is trying to pick up some lawn mowing jobs over the weekend to make extra money for a school trip. Each lawn in his neighborhood takes an average of 40 minutes to mow, and Aiden has no more than 11 hours, or 660 minutes, of available time to mow lawns. If Aiden mows his grandparents' farm which takes him 110 minutes, and x represents the number of lawns he mows in his neighborhood, which inequality represents this situation?
A.
40x + 110 ≤ 660
B.
110x + 40 ≤ 660
C.
110x + 40 ≥ 660
D.
40x + 110 ≥ 660

Answers

Answer:

A.

Step-by-step explanation:

40x is the number of lawns he can do, less the time to do his grandparents time (added to other law time) and he has 660 mins of less to complete them.

Answer:

a. 40x + 110 ≤ 660

Step-by-step explanation:

Solve the system of linear equations and check any solutions algebraically.

Answers

Answer:

[tex]\boxed{\sf \ \ x = 9, \ y = -5, \ z = 5 \ \ }[/tex]

Step-by-step explanation:

Hello,

   (1) 2x + 4y + z = 3

   (2) x - 2y - 3z = 4

   (3) x + y - z = -1

From (3) we can write z = x + y + 1 and we replace in (1)

2x + 4y + x + y + 1 = 3 <=> 3x + 5y = 3-1 =2

   (1') 3x + 5y = 2

and we replace in (2)

x - 2y -3(x+y+1) = 4 <=> -2x -5y -3 = 4 <=> -2x -5y = 4 + 3 = 7

   (2') -2x - 5y = 7

(1') + (2') gives

3x - 2x + 5y - 5y = 2 + 7 = 9

x = 9

we replace in (1')

3*9 + 5y = 2 <=> 27 + 5y = 2 <=> 5y = 2-27 = -25 <=> y = -25/5 = -5

y = -5

and then in (3)

9 - 5 - z = -1 <=> 4 - z = -1 <=> z = 4 + 1 = 5

z = 5

hope this helps

Answer:

work is shown and pictured

6th grade math, help me please

Answers

Answer:

1. 2/5

Step-by-step explanation:

When it says the ratio is 5 to 2, that means 5 is always first:

5 : 2 is correct

5/2 is correct

10 : 4 is correct (multiplied 2 on both sides)

2/5 is incorrect because 2 is first. That means that this ratio would be 2 to 5, not 5 to 2.

Answer:

2/5

Step-by-step explanation:

because you are not supposed to flip the two numbers. You need to keep them in the same order.

Sorry, if this isn't the greatest answer. Its my first time.

In the figure below, YZA and YZX are right angles, XYZ and AYZ are congruent, and XZ = 10. What is the length of ?



A.
25

B.
20

C.
10

D.
5

Answers

Answer:

  C.  10

Step-by-step explanation:

The given information tells you that triangles YZX and YZA are congruent, so ZA = ZX = 10.

Please show step by step working out of stationary points and points of inflection with the y coordinates (and sketch graph) for the equation y=x^4-36x^2​

Answers

Answer:

See picture attached

Step-by-step explanation:

Given the radius of a circle is 7 cm, what is the circumference?

Answers

Answer:

14π or 43.96

Step-by-step explanation:

C = 2πr and we know that r = 7 so C = 14π or 43.96.

F(n)=6.5n+4.5 find the 5th term of the sequence defined by the given rule

Answers

Answer:

37

Step-by-step explanation:

To find the fifth term , we have to take the value of n as 5

So, F(5)= 6.5 (5) +4.5

= 32.5 + 4.5

= 37

According to medical data, the ages at which patients have their first knee replacement surgery
follows a normal distribution. The average age for a first knee replacement is 58 years of age, with a
standard deviation of 8.25 years. Therefore, doctors can expect the middle 68% of their knee
replacement surgery patients to be between what ages?

Answers

Answer:

The doctors can expect the middle 68 % of their knee replacement surgery patients to be between 49.75 years and 66.25 years.

Step-by-step explanation:

68 % of the knee replacement surgery patients implies that the ages lies within x = x₀ ± σ where x₀ = mean age = 58 years and σ = standard deviation = 8.25 years

So, the ages lies between x₀ + σ and x₀ - σ

So, the ages lie between 58 - 8.25 = 49.75 years

and 58 + 8.25 = 66.25 years

So the doctors can expect the middle 68 % of their knee replacement surgery patients to be between 49.75 years and 66.25 years.

A factory manufactures chairs and tables, each requiring the use of three operations: cutting, assembly, and finishing. The first operation can use at most 40 hours; the second at most 42 hours; and the third at most 25 hours. A chair requires 1 hour of cutting, 2 hours of assembly, and 1 hour of finishing; a table needs 2 hours of cutting, 1 hour of assembly, and 1 hour of finishing. If the profit is $20 per unit for a chair and $30 for a table, what is the maximum revenue? Round your answer to the nearest whole number. Do not include a dollar sign or comma in your answer.

Answers

Answer:

z(max) = 650 $

x₁ = 10 units

x₂ = 15 units

Step-by-step explanation:

That is a linear programming problem, we will use a simplex method to solve it

Formulation:

Let´s call  x₁  number of chairs   and x₂ number of tables then :

Item              (in hours)     cutting       assembly      finishing        Profit ($)

Chairs (x₁)                              1                   2                     1                      20

Tables (x₂)                              2                   1                      1                      30

Availability                           40                 42                   25

Objective Function

z  =  20*x₁   +  30x₂   ( to maximize) subject to:

x₁  +  2x₂   ≤  40

2x₁  + x₂    ≤  42

x₁ + x₂     ≤    25

x₁  ,   x₂  >= 0

Using excel or any other software we find:

z(max) = 650

x₁ = 10

x₂ = 15

The chairs and tables manufactured by the factory is an illustration of linear programming, where the maximum revenue is 674

Let x represent chairs, and y represent tables

So, the given parameters are:

Cutting:

Chairs: 1 hourTable: 2 hoursHour available: 40

So, the constraint is:

[tex]\mathbf{x + 2y \le 40}[/tex]

Assembly:

Chairs: 2 hoursTable: 1 hourHour available: 42

So, the constraint is:

[tex]\mathbf{2x + y \le 42}[/tex]

Finishing:

Chairs: 1 hourTable: 1 hourHour available: 25

So, the constraint is:

[tex]\mathbf{x + y \le 25}[/tex]

The unit profit on the items are:

Chairs: $20Table: $30

So, the objective function to maximize is:

[tex]\mathbf{Max\ z = 20x + 30y}[/tex]

And the constraints are:

[tex]\mathbf{x + 2y \le 40}[/tex]

[tex]\mathbf{2x + y \le 42}[/tex]

[tex]\mathbf{x + y \le 25}[/tex]

[tex]\mathbf{x,y \ge 0}[/tex]

Using graphical method (see attachment for graph), we have the following feasible points:

[tex]\mathbf{(x,y) = \{(10,15),\ (17,8),\ (14.67, 12.67)\}}[/tex]

Calculate the objective function using the feasible points.

[tex]\mathbf{z = 20 \times 10 + 30 \times 15}[/tex]

[tex]\mathbf{z = 650}[/tex]

[tex]\mathbf{z = 20 \times 17 + 30 \times 8}[/tex]

[tex]\mathbf{z = 580}[/tex]

[tex]\mathbf{z = 20 \times 14.67+ 30 \times 12.67}[/tex]

[tex]\mathbf{z = 673.5}[/tex]

Approximate

[tex]\mathbf{z = 674}[/tex]

Hence, the maximum revenue is 674

Read more about linear programming at:

https://brainly.com/question/14225202

What is the image of the point by (-5,3) under a 270 rotation about the point (-7,-3)

Answers

Step-by-step explanation:

here, the given point is (-7,-3)

now, by the formula,

p(x,y)= p-1 (-y+a+b,x-a+b) ( p-1 is p das)

p(-5,3)= p-1 (-13,-1) is answer.

hope it helps..

A political analyst predicts Mr. Smith will only get 122 votes for mayor. If Mr. Smith only gets 57 votes, what is the political analyst's percent error?

Answers

Answer:

65%

Step-by-step explanation:

what equals 1+1= Why can't I see any answers help i logged off etc is it just me?

Answers

Answer:

1 + 1 = 2

Step-by-step explanation:

^

Answer:

no , it's happening to everyone , even I can't see it .

The sum of a number and 9 is subtracted from 60. The result is 10. Find the number.

Answers

Answer:

Number : 41

Step-by-step explanation:

Say that this number is x. The sum of this number ( x ) and 9 subtracted from 60 will be 10. Therefore we can create the following equation to solve for x,

60 - (x + 9) = 10,

60 - x - 9 = 10,

51 - x = 10,

- x = 10 - 51 = - 41,

x = 41

This number will be 41

Consider a triangle ABC like the one below. Suppose that B=36°, C= 62°, and b= 40. (The figure is not drawn to scale.) Solve the triangle.
Round your answers to the nearest tenth.
If there is more than one solution, use the button labeled "or".

Answers

Answer:

A=82°

a= 67.4

c = 60.1

Step-by-step explanation:

For A

A+B+C =180°

A= 180-(B+C)

A= 180-(36+62)

A= 189-(98)

A= 82°

For a

a/sinA= b/sinB

a/sin82= 40/sin36

a= (40*sin82)/sin36

a=( 40*0.9903)/0.5878

a=67.39

Approximately = 67.4

For c

c/sinC= b/sinB

c= (sinC*b)/sinB

c= (sin62*40)/sin36

c =(0.8829*40)/0.5878

c = 60.08

Approximately = 60.1

help i need to know pls

Answers

Answer:

7.8 =x

Step-by-step explanation:

Since this is a right triangle, we can use trig functions

tan theta = opp / adj

tan 48 = x/7

7 tan 48 = x

7.774287604 = x

To the nearest tenth

7.8 =x

Type the correct answer in the box. Use numerals instead of words. What is the missing value in the inverse variation given in the table?

Answers

Answer:

48

Step-by-step explanation:

If x varies inversely as y, we have:

[tex]x \propto \frac{1}{y} \\\implies x = \frac{k}{y}[/tex]

When x=2, y=96

[tex]2 = \frac{k}{96}\\k=192[/tex]

When x=8, y=24

[tex]8 = \frac{k}{24}\\k=192[/tex]

Therefore, the constant of proportionality, k=192.

The equation connecting x and y is:

[tex]x = \frac{192}{y}[/tex]

When x=4

[tex]4 = \frac{192}{y}\\4y=192\\y=48[/tex]

The missing value in the inverse variation given in the table is 48.

Look at the number pattern shown below:3 × 17 = 5133 × 167 = 5511333 × 1667 = 555111What will be 33333 × 166667?

Answers

Answer:

33333 x 166667 = 5555511111

I think that is the answer you wanted

Step-by-step explanation:

      166667

     x 33333

5555511111

HELP PLEASE FOR 35 POINTS!!!! Solve the rational equation 3 divided by x equals quantity 4 times x plus 3 divided by x squared, and check for extraneous solutions.

Answers

Answer:

[tex]x=-3[/tex]

Step-by-step explanation:

So, we are given:

[tex]\frac{3}{x}=\frac{4x+3}{x^2}[/tex]

First, we should immediately rule out 0 as an answer. This is because the if [tex]x=0[/tex], the equation would be undefined.

[tex]x\neq 0[/tex]

Now, cross multiply.

[tex]3(x^2)=x(4x+3)[/tex]

[tex]3x^2=4x^2+3x[/tex]

Divide everything by x (and we can do this safely because we already know x cannot be equal to zero).

[tex]3x=4x+3[/tex]

[tex]-x=3[/tex]

[tex]x=-3[/tex]

We didn't run into any possibilities for extraneous solutions.

Examine today’s stock listing for SFT Legal, shown below. 52 wk High 52 wk Low Symbol Div. Close Net Change 74.80 44.61 SFT 8.94 56.11 5.74 What was the price of SFT Legal yesterday? a. $47.17 b. $56.11 c. $50.37 d. $61.85

Answers

Answer:

c. $50.37

Step-by-step explanation:

Close price was $56.11 and net change was $5.74. so subtract the net change from the close to get yesterday's price.

Answer:

c.50.37

Step-by-step explanation:

A catering company is catering a large wedding reception. The host of the reception has
asked the company to spend a total of $454 on two types of meat: chicken and beef. The
chicken costs $5 per pound, and the beef costs $ 7 per pound. If the catering company
buys 25 pounds of chicken, how many pounds of beef can they buy?

Answers

The answer is 47 pounds

Explanation:

1. First, let's calculate the amount of money that was spent on chicken

$5 per pound of chicken x 25 pounds = $125

2. Calculate the amount of money left to buy beef by subtracting the total spend on chicken to the total of the budget.

$454 (total) - $125 (chicken)  =  $329

3. Calculate how many pounds of beef you can buy with the money left by dividing the money into the price for one pound.

$329 / $7 = 47 pounds

A bag contains a collection of distinguishable marbles. The bag has two red marbles, three green ones, one lavender one, two yellows, and two orange marbles. HINT [See Example 7.] How many sets of four marbles include exactly two green marbles

Answers

Answer:

63

Step-by-step explanation:

Given that;

The bag has two red marbles,  n(red) =2

three green ones marbles,  n(green) = 3

one lavender one marbles,  n(lavender) = 1

two yellows marbles,            n(yellow ) = 2

two orange marbles.              n(orange) = 2

number of non green marbles = 2+1+2+2 = 7

The objective is to find out how many sets of four marbles include exactly two green marbles

Since sets of four marbles contain exactly two green marbles, then N(select 2 from 3 marbles and 2 from 7 marbles)

= [tex]^3C_2 \times ^{7}C _2[/tex]

= [tex]\dfrac{3!}{2!(3-2)!} \times \dfrac{7!}{2!(7-2)!}[/tex]

= [tex]\dfrac{3*2!}{2!} \times \dfrac{7*6*5!}{2!(5)!}[/tex]

=  [tex]3 \times 7\times 3[/tex]

= 63

Amber says that the data set is left-skewed because the box is farther to the left on the number line. (A) Is Amber correct? (B) Explain your reasoning.

Answers

A distribution that is skewed left has exactly the opposite characteristics of one that is skewed right: the mean is typically less than the median; the tail of the distribution is longer on the left hand side than on the right hand side; and. the median is closer to the third quartile than to the first quartile.

Help, Please. File attached this time. Sorry

Answers

Answer:

27 degrees

Step-by-step explanation:

Because there is a straight line, all of the angles shown add up to 180, so if you add up all the given angles and subtract it from 180, you will get your answer.

39+86+28=153

180-153=27

Hope this helps. If you have any follow-up questions, feel free to ask.

Have a great day! :)

helpppppppppppppppppppppppppppppppppppppppppppppppppppppp

Answers

Answer:

4

Step-by-step explanation:

Answer:

1/8 < 1/6

Step-by-step explanation:

The top is divided into 8 and 1 part is shaded so 1/8

The bottom is divided into 6 and 1 part is shaded so 1/6

Comparing

1/8 < 1/6

Roberta sold goods costing $35,500, her expenses totaled $2,500 and her freight in totaled $750.
Her company's average stock of goods during the same period was $9,500.
The inventory turnover ratio for Roberta's company is

Answers

Answer:

Inventory turnover ratio is 3.74

explanation:

Inventory turnover is a ratio of the number of times a company's inventory is sold and replaced in a given period.

Inventory turn over ratio is calculated as ; Cost of goods sold ÷ Average stock of goods sold

= $35,500 / $9500

= 3.74

the solution of the equation 0=4+4(m+1) is​

Answers

Answer:

[tex]\boxed{m = -2}[/tex]

Step-by-step explanation:

[tex]0 = 4+4(m+1)[/tex]

Resolving Parenthesis

[tex]0 = 4+4m + 4[/tex]

[tex]0 = 4m+8[/tex]

Subtracting 8 to both sides

[tex]-8 = 4m[/tex]

[tex]4m = -8[/tex]

Dividing both sides by 4

m = -8/4

m = -2

Step-by-step explanation:

4+4m+4= 0

4m+8=0

4m=-8

m= -8/4=-2

Graph a line that contains the point (-7,-4)and has a slope of - 2/3

Answers

Hi there! :)

Answer:

Given the information, we can write an equation in slope-intercept form

(y = mx + b) to graph the line:

Plug in the slope for 'm', the y-coordinate of the point given for 'y', and the

x-coordinate given for 'x':

-4 = -2/3(-7) + b

-4 = 14/3 + b

Solve for b:

-12/3 = 14/3 + b

-12/3 - 14/3 = b

-26/3 = b

Therefore, the equation of the line is y = -2/3x - 26/3 (Graphed below)

Some points on the line include:

(-7, -4)

(-4, -6)

(0, -26/3)

(2, -10)

(5, -12)

20
When converting from inches to feet, the measurement in inches, m, of an object varies directly with its measurement
in feet, f, with the constant of variation being 12.
What is the equation relating these two quantities?

Answers

Answer:

[tex]m=12f[/tex]

Step-by-step explanation:

The measurement in inches =m

The measurement  in feet = f

We are told that: m varies directly with f

Written mathematically:

[tex]m\propto f[/tex]

Introducing the constant of variation, we have:

[tex]m=k f[/tex]

Given that: k=12

The equation relating these two quantities is:

[tex]m=12f[/tex]

Other Questions
during which stage of a collaborative discussion should a person acknowledge key points others' statements and ask necessary clarifying questions Tasha wants to find out if she is going to pass her test. She decides that she willsimulate her test by flipping a coin. This meansa) There is a 50-50 chance she will pass her test.b) All of the questions are multiple choice.c) She hasn't studied.d) There is a 20% chance she will pass. horacio y fernando ___ en la cafeteria cuando miguel se cayo al suelo interact to produce a trait in which neither dominant gene can produce its effect in the presence of the other.O Multiple allelesChromosomesO CarriersSex linked genes Your birthday was in June and your rich Aunt Edna, who had been acting a little weird lately, gave you a check for $25,000 to help you celebrate. You opened a checking account with this money by making a DEPOSIT on June 16th and proceeded to have the time of your life!The first thing you did was move out of your parents house and into a high-rise apartment near downtown. On June 25th, you wrote CHECK #101 to the Trumble Tower for $1800 which included a $600 deposit. You wanted your new flat to be really cool, so you decided to decorate it with style and taste. You bought a home theater system and television at Best Buy using your Mastercard. At Rooms-To-Go, you purchased a living room set and bedroom set using your VISA card. You used your DEBIT card on July 4th for a $129.45 purchase at Macys. You invited all your friends over the following Saturday night for pizza and drinks and paid for it with your Mastercard. On July 8th you realized that you needed some food and supplies for a day at the beach. First you went to Starbucks for breakfast using your DEBIT card to pay the $12.83 bill. You stopped at Walmart and bought $132.56 in food and supplies using your DEBIT card. At Krogers you used your DEBIT card to purchase $52.23 of groceries and your VISA card to buy gas. You also stopped by the ATM machine at your bank to withdraw $100 for the day at the beach.On July 15th, you filled out a DEPOSIT slip for your paycheck of $250 since you forgot to sign up for direct deposit. After driving to and from the beach, you realized you needed a new car! You decided to go car shopping on July 16th since you did not have to work that day. At Best Friend Ford, you found the perfect car for $12,498.99! You wrote CHECK #102 for the car. Since you got a new car, you decided that you needed to look good while driving it so you went shopping on July 19th. You purchased clothing at H&M and Abercrombie & Fitch using your Mastercard.Since you had been driving around in a new car wearing new clothes, your friends decided that they wanted to be paid the money that you owed them. Paul Smith accepted the $20 CHECK #103 you wrote him on July 21st. Linda Boltman and Max Badger wanted cash immediately so you went to the nearest gas station with an ATM and withdrew $80 in cash to pay them on July 23rd. Your bank charged you an ATM service FEE of $2.50 for the withdrawl.At the end of the month, the bills started to arrive. You forgot to set up online banking with your checking account so you had to write checks. You paid the utility company Greenhouse Energy with CHECK #104 for $155.87 on July 25th, paid AllConnect with CHECK #105 for $150 for TV, internet, and phone on July 27th, paid Mastercard with CHECK #106 for $1518.23 on August 3rd, and paid VISA with CHECK #107 for $1994.02 on August 5th. You wrote a DEPOSIT slip on August 1st for your $250 paycheck and got $30 cash back with the deposit.THE ENDCheck Register CodesATM ATM WithdrawalDEB Debit CardDEP DepositFEE Checking Account Fee### (check number) Check2 what is a width of a rectangle prisim if the volume is 50046 cm 3 the hight is 7 cm and the llength is 13 cm please help :) What is 7.7 x 10 to the 8 power written in standard form? A. 770,000,000 B. 77,000,000,000 C. 77,000,000 D. 7,700,000,000 Question 5 of 15Water turns from a gas to a liquid in a process calledA. evaporationB. sublimationc. transpositionD. condensationSUBMIT 1) What was the effect of Roe v. Wade? A) The ruling provided that those accused of a crime be provided counsel even if they are unable to afford a lawyer. B) The ruling determined the power established by the Commerce clause to belong to the federal government. C) The ruling included symbolic speech under the free speech rights of the First Amendment. D) The ruling was hailed by feminists as a major step in guaranteeing individual civil rights. 2) What efforts were made by President Lyndon B. Johnson to improve American society? A) President Johnson endorsed national reforms to improve conditions for those in poverty. B) President Johnson sought to end the Vietnam War through gradually removing troops. C) President Johnson established government work programs to reduce the unemployment rate. D) President Johnson reduced taxes on the wealthy believing it would improve the economy. 3) What was the result of President Carters diplomacy between Israel and Egypt in 1978? A) the dissolution of the Geneva Convention B) the signing of the Camp David Accords C) the Yalta Conference in Jerusalem D) the Kyoto Protocol ending the Six-Day War 4) Which statement most accurately examines how President Obama influenced health care during his first term? A) He extended the reach of Medicaid to millions of Americans to create a free universal health care system. B) He established a federal health care mandate that would provide low-income Americans with health care. C) He eliminated privatized health insurance and established a socialized health care system run by the government. D) He required medical insurance companies to reduce their rates so that more Americans could afford health insurance. 5) What was an effect of President George W. Bushs Ownership Society on the United States? A) Bushs Ownership Society finished the objectives set forth by President Johnsons War on Poverty. B) Elimination of competition created monopolies in the banking industry until the economic collapse in 2008. C) The easy affordability of home loans created risky loans that would lead to an economic recession. D) The creation of more homeowners by Bushs initiatives created more small businesses and eliminated chain stores. 6) Which US foreign policy was focused on containing the spread of communism? A) Truman Doctrine B) Roosevelt Corollary C) isolationism D) imperialism 7) Which is one reason that President Truman decided to use the atomic bomb on Japan? A) Stalin warned Truman that he would pull Soviet support for the Allies if the United States did not drop the bomb. B) The United States wanted to show Nazi Germany the power of the bomb to get their surrender as well. C) It was Franklin Roosevelts dying request to his vice president. D) Kamikaze pilots proved that Japanese soldiers would not surrender unless ordered to do so. 8) Which statement most accurately defines an alteration of the presidents power due to the Vietnam War? A) Congress passed Executive Order 11255 to designate Vietnam as an international combat zone. B) The Supreme Court declared in United States v. Johnson that the president needed to submit a quarterly report to Congress on the progress of the war. C) A resolution was passed making it mandatory for the president to consult Congress before initiating conflict. D) Congress ratified the Twenty-Fifth Amendment, which defines the succession of the president due to illness or death. 9) How did the launching of Sputnik I affect the United States? A) It led to the development of the hydrogen bomb. B) It led to the creation of NASA. C) It led to the development of the domino theory. D) It led to the creation of HUAC. 10) How was commerce changed in North America during the Clinton administration? A) NAFTA created incentives for North American businesses to move overseas due to the restrictions on the movement of goods. B) NAFTA allowed for the free movement of citizens of all three countries throughout North America. C) NAFTA provided for easier transportation and lower tariffs of goods and services throughout North America. D) NAFTA increased the price of goods in the United States and raised taxes in Canada and Mexico. A vat of nitrogen at its boiling point (-196 degrees C) absorbs 384000 J of heat. How much mass of nitrogen burns off? (Unit = kg) Three books are at rest, in equilibrium, on a horizontal table asshown. The weight of each book, which is equal to the forcegravity exerts in the downward direction, is given. What is thenet force on the middle book? Etsuro is a management consultant. Baker Corp. asks him to evaluate their company, and he finds that the difference between the cost of producing the firm's products and the value of those products is extremely narrow. What should Etsuro suggest that Baker Corp. management do? Find a way to pass on as much profit as possible to suppliers and customers. Find a way to widen the gap between cost and value. Shore up the company's strong position by erecting entry barriers. Encourage customers to buy complements to their products. Offshore oil-drilling operations entail an unavoidable risk of an oil spill, but importing oil on tankers presently entails an even greater such risk per barrel of oil. Therefore, if we are to reduce the risk of an oil spill without curtailing our use of oil, we must invest more in offshore operations and import less oil on tankers. Which of the following, if true, most seriously weakens the argument above. A) Tankers can easily be redesigned so that their use entails less risk of an oil spill. B) Oil spills caused by tankers have generally been more serious than those caused by offshore operations. C) The impact of offshore operations on the environments can be controlled by careful management. D) Offshore operations usually damage the ocean floor, but tankers rarely cause such damage. E) Importing oil on tankers is currently less expensive than drilling for it offshores. A lichen is an organism that structurally appears to be a single organism. But a lichen is actually two different organismsa fungus and green algaeliving together as one organism. The fungal partner derives its nutrition from the photosynthesizing algae. How does a lichen differ in its photosynthetic activity from Elysia chlorotica, the sea slug thats considered to be a photosynthesizing animal? A. In lichens, the fungi photosynthesize on their own, while Elysia chlorotica forms a relationship with a photosynthesizing plant. B. In lichens, the relationship with the algae lasts throughout the life cycle, but in Elysia chlorotica, the relationship occurs only during the immature juvenile stage of the slugs life cycle. C. Lichens can photosynthesize only because of the living algal partner, while Elysia chlorotica incorporates chloroplasts from algae into its cells. D. In lichens, the association between the fungi and algae is permanent, while Elysia chlorotica associates with a photosynthesizing organism only when it requires food. There are seven roads that lead to the top of a hill. How many different ways are there to reach the top and then go back down? Which of the following characteristics cannot be influenced by the environment? A. Chance of having diabetes B. Metabolism C. Natural eye color D. Height PLEASE HELP The probability distribution for arandom variable x is given in the table. Please help. Ill mark you as brainliest if correct! Las fotos de Gabriel Gabriel has met a new friend in his Spanish class. Now he is telling her about his life in the United States and is showing her some pictures of his family and friends. Complete Gabriels description using the most appropriate forms of ser or estar. ( Quieres ver mis fotos, Amalia? Esta persona aqu (1) _____ mi amigo John. l (2) _____ de Arizona. En esta foto, mis amigos y yo (3) ______ en una playa que (4) _______ cerca de San Diego. Y aqui ves mi mam. Ella (5) __es___ muy amable. Mi mam (6) _____ mdico en un hospital muy grande. Oh en Esta foto, yo (7) _______ ocupado limpiando mi habitacin porque (8) _______ muy sucia. Mira la cara de mis padres, ellos (9) ______ muy enojados conmigo! Mira el aeropuerto de Los ngeles, aqu yo (10) ________ preparado para tomar el avin a San Salvador. Extrao mucho mi vida en los Estados Unidos the sum of two consecutive multiples of 5 is 55.what are the multiples